MathGroup Archive 2002

[Date Index] [Thread Index] [Author Index]

Search the Archive

Pb Limit ArcTan to : - Infinity

  • To: mathgroup at smc.vnet.net
  • Subject: [mg34624] Pb Limit ArcTan to : - Infinity
  • From: "Didier Losfeld" <Didier.Losfeld at hei.fr>
  • Date: Thu, 30 May 2002 02:55:04 -0400 (EDT)
  • Sender: owner-wri-mathgroup at wolfram.com


Bonjour ,


I do not understand answers of Mathematica :

question 1 : Series [ ArcTan [ x ] , { x , - Infinity , 2 } ]

answer : Pi / 2 - 1 / x + O ( 1 / x ) ^ 3

question 2 : Limit [ ArcTan [ x ] , x -> - Infinity ]

answer : - Pi / 2

The first answer is False , the second answer is correct.

Many thanks in advance.

Didier Losfeld Professor of mathematics ( France )





  • Prev by Date: Extraction of positive values from list of both +/- values
  • Next by Date: Mathematica graphs in PowerPoint
  • Previous by thread: RE: Extraction of positive values from list of both +/- values
  • Next by thread: Re: Pb Limit ArcTan to : - Infinity